Vous êtes sur la page 1sur 42

M.K.

GUPTA
CA EDUCATION
PH: 9212011367/ 9811429230 www.mkguptacaclasses.com
e-mail id: mkguptataxclasses@rediffmail.com

SERVICE TAX
25 MARKS

CA
IPCC
MAY-2013/ NOV-2013 F.Y. 2012-13 F. A. 2012 `100

Service Tax

SERVICE TAX
INDEX

FINANCE ACT 1994


Extent, Commencement and Application Definition Charge of Service Tax Negative List of services Bundled Services Valuation of taxable services for charging service tax Date of determination of rate of tax, value of taxable service and rate of exchange Payment of service tax Registration Furnishing of returns Scheme for submission of returns through service tax returns preparers Service tax collected from any person to be deposited with Central Government Interest on amount collected in excess Interest on delayed payment of service tax Penalty for contravention of rules and provisions of Act for which no penalty is specified elsewhere. Power to grant exemption from service tax Power to grant rebate (Duty drawback) in case of exports Power to make rules

SECTIONS 64 65B 66B 66D 66F 67 67A 68 69 70 71 73A 73B 75 77 93 93A 94 Rule 2 4 5 6 7

SERVICE TAX RULES, 1994


Reverse Charge Registration Records Payments Return

POINT OF TAXATION RULES, 2011


Date of payment Point of Taxation for Payment of Service Tax Point of Taxation for change in rate of service tax Point of Taxation for Service Tax on New Services Payment of Service Tax on Receipt Basis in certain cases Determination of point of taxation in case of copyrights, etc.

Rule 2A 3 4 5 7 8

Service Tax

APPLICABILITY/NON-APPLICABILITY OF NEGATIVE LIST AND RELATED TOPICS IN SYLLABUS OF PART II: SERVICE TAX AND VAT OF PAPER 4: TAXATION (IPCC) FROM MAY 2013 EXAMINATION ONWARDS The Examination Committee at its 498th meeting held on September 23, 2012 has decided that the following sections of the Finance Act, 1994/rules will be covered under point no. 1 (Concepts and General principles) and point no. 2 (Charge of service tax and taxable services) of syllabus of Part II: Service tax & VAT in Paper 4: Taxation from May 2013 examination onwards: (i) Section 64 - Extent, commencement and application (ii) Sections 65B Interpretations [only relevant ones which would be required to explain the concepts included] (iii) Section 66B Charge of service tax on and after Finance Act, 2012 (iv) Section 66D - Negative list of services (v) Section 66F Principles of interpretation of specified descriptions of services or bundled services. (vi) Section 67A - Date of determination of rate of tax, value of taxable service and rate of exchange (vii) Point of Taxation Rules, 2011 It may be noted that the above-stated Sr. No.(i) - Section 64 and Sr. No.(vii) - Point of Taxation Rules, 2011 have already been covered in the existing syllabus so far. Thus, only Sr. Nos.(ii) to (vi) have been included as new topics in the syllabus under point no.1 (Concepts and General principles) and point no.2 (Charge of service tax and Taxable services). The Committee further decided that the following topics pertaining to service tax law will not be included in the syllabus: (i) Section 66C - Determination of place of provision of service (ii) Section 66E Declared services (iii) Place of Provision of Service Rules, 2012 (iv) Section 68(2) and Reverse Charge notification (v) Export of Services vide rule 6A of the Service Tax Rules, 1994 (vi) Mega exemption notification and other exemptions (vii) Abatement notification In furtherance to the above-mentioned decision, it is clarified that other topics viz., valuation of taxable service, payment of service tax and filing of returns, topics related to VAT etc. as covered in the Study Material will continue to apply as they used to apply before. In view of the far-reaching changes effected with the introduction of negative list approach of taxation in service tax, Study Material and Practice Manual of Part-II: Service tax & VAT of Paper 4: Taxation (contained in Volume III) [November edition] have been completely revamped. Students may note that the revised Study Material and Practice Manual have been prepared in accordance with the abovementioned decision of the Examination Committee. In other words, all the chapters of the revised Study Material and Practice Manual would be applicable for examination purposes. Hence, students with the older editions of the Study Material and Practice Manual are advised to read the new edition [November edition].

Service Tax

Service Tax in India The genesis of service tax is the result of recommendations made in 1990s by the Tax Reforms Committee headed by Professor Dr. Raja J. Chelliah. Based on the above recommendations, Dr. Manmohan Singh, the Union Finance Minister, introduced the concept of service tax in 1994-95 w.e.f. 01.07.1994. Initially service tax was imposed only on three services i.e. telephones, non-life insurance and stock brokers but at present all the services are taxable except the services covered in the negative list given under section 66D. Certain services covered in the notification no. 25/2012 dated 20.06.2012 are also exempt from service tax (and are called Mega Exemptions) Question 1: Explain Selective Approach/ Comprehensive Approach to Service Tax. Answer: Each country has a taxation system on services adopting either a Comprehensive Approach or a Selective Approach. In Comprehensive Approach all services are made taxable and a list is given in case some services are to be exempted. In Selective Approach, selective services are subjected to service tax. While most of the developed countries tax all the services with very few and limited exemptions, most of the developing countries have opted for taxation of select services only. India has adopted a selective approach in the beginning but from financial year 2012-13, there is comprehensive approach i.e. now all services are taxable except the services given in the negative list as per section 66D. Question 2 (Imp.): Write a note on Constitutional validity of Service Tax. Answer: The Government derives its power to levy taxes from Constitution of India. As per article 265, no tax shall be levied or collected except by authority of law. As per Article 246, the Constitution in its Schedule VII has mentioned the matters on which Central Government and State Government can make laws. Such matters are divided into three categories. List I: Union List (It contains the matters in respect of which only the Central Government can make law). (Total Entries 97) List II: State List (It contains the matters in respect of which only the State Government can make law). (Total Entries 66) List III: Concurrent List (It contains the matters in respect of which both the Central Government and the State Government have power of legistration). (Total Entries 46) Initially there was no specific entry in the Union List for levying service tax. Service tax was levied by the Central Government by drawing power from Entry 97 of the Union List. Entry 97 is a residuary entry in List-I, which has been reproduced below: 97 Any other matter not enumerated in List II or List III including any tax not mentioned in either of those Lists. The residuary entry provides wide power to the Central Government in respect of taxation of the subjects not mentioned in the Lists given by the constitution. Entry 92C was introduced in the VII schedule in the Union List with effect from 07.01.2004 vide article 268A. Entry 92C reads as under: 92C Taxes on services But till date this entry (92C) is kept dormant (ineffective).

Service Tax Question 3 (V. Imp.): Write a note on sources of Service Tax Law. Answer: Sources of Service Tax Law are as given below: 1. Finance Act, 1994 2. Rules 3. Notifications 4. Circulars 5. Trade Notices

1. Finance Act, 1994 The provisions relating to service tax are given in chapter V of Finance Act, 1994. Later, Finance Act, 2003 inserted chapter VA in Finance Act, 1994. It contains section 64 to 96-I. In the subsequent years additional provisions were introduced. So far there is no Service Tax Act. In the year 2004, the provisions relating to charging of education cess on the service tax were made applicable through Chapter VI of the Finance (No.2) Act, 2004. Thereafter secondary and higher education cess was levied through Chapter VI of Finance Act, 2007. 2. Rules on Service Tax Section 94/96-I of Finance Act, 1994 grants power to the Central Government to make rules for service tax. Central Government has made rules as given below: (i) (ii) (iii) (iv) (v) (vi) (vii) Service Tax Rules, 1994 Point of Taxation Rules, 2011 Service Tax (Determination of Value) Rules, 2006 Service Tax Return Preparer Scheme, 2009 Service Tax (Registration of Special Category of Persons) Rules, 2005 CENVAT credit Rules, 2004 Place of Provision of Services Rules, 2012

3. Notifications on Service Tax Section 93, 94, 96-I of Finance Act, 1994 empowers Central Government to issue notifications to exempt any service from service tax. 4. Circulars and office letters on Service Tax The Central Board of Excise and Custom issues circular and instruction to explain scope of taxable services and the scheme of service tax administration. The circulars bring out the real intention of the legislature. 5. Trade Notices on Service Tax Trade notice are issued by the commissioners for effective administration of service tax law. The trade notice are circulated among the field officers and also copies are given to Trade Associations. Question 4: Write a note on Administration of Service Tax. Answer: Ministry of Finance has two main Departments 1. Department of Revenue 2. Department of Expenditure All the taxes are covered under the Department of Revenue and the Department of Revenue has two Board of Taxes: 1. Central Board of Direct Taxes 2. Central Board of Excise and Customs Service Tax is administered by CBEC and the administration of service tax is as given below in the descending (decreasing) order: 1. Central Board of Excise and Customs 2. Chief Commissioners of Central Excise

Service Tax

3. Commissioners of Central Excise [Director General of Service Tax (Co-ordinator between 1 & 3)] The work relating to service tax is also under the control of CBEC, however in 1997 a new post of Director General (Service Tax) was formed. The functions and power of Direct General (Service Tax) are as follows: (1) To monitor the collection and assessment of service tax. (2) To study the staff requirement at field level. (3) To increase revenue collection or to streamline procedures. (4) To undertake study of law and procedures in relation to service tax with a view to simplify the service tax collection and assessment and make suggestions thereon. (5) To form a data base regarding the collection of service tax. (6) To inspect the service tax cells in the Commissionerate. (7) To undertake any other functions as assigned by the Board from time to time. Question 5: Write a note on Service Tax Procedures. Answer: Service tax procedures includes (i) Registration (ii) Maintenance of books and records (iii) Payment of service tax (iv) Availment and utilization of CENVAT credit (v) Filing of service tax returns (vi) Assessment and recovery of service tax (vii) Interest and penalties (viii) Rectification of mistakes (ix) Revision of assessment order (x) Appeals (xi) Search and seizure (xii) Advance rulings etc. Question 6 (Imp.): Write a note on role of a Chartered Accountant in service tax. Answer: A Chartered Accountant is well equipped to position himself in the new role as an advisor and facilitator for due compliance of service tax law. The nature of services are: 1. Advisory services: A Chartered Accountant is considered to be qualified and competent to comprehend (understand) and interpret the service law to give required advice and guidance. 2. Procedural compliance: The service tax law requires Registration, Payment of tax, Filing of returns etc. A Chartered Accountant is well suited to assist the assessee in all the above functions and ensure compliance. 3. Personal representation: A Chartered Accountant is allowed to appear before the assessment authority, Commissioner (Appeals) and Custom, Excise and Service Tax Appellate Tribunal. With his expertise a

Service Tax Chartered Accountant can represent his clients.

4. Certification and audit: Service tax Returns and Financial Statements can be certified by the Chartered Accountant. Question 7 (V. Imp.): Write a note on Extent, Commencement and Application of Service Tax. Answer: Extent, Commencement and Application Section 64 The Finance Act, 1994 came into force from 01.07.1994. The Act extends to the whole of India except the state of Jammu and Kashmir. Thus, services provided in the state of Jammu and Kashmir are not liable to service tax. As per Article 370 of the Constitution, any Act of Parliament applies to Jammu & Kashmir only with concurrence (approval) of State Government. Since, no such concurrence has been obtained in respect of Finance Act, 1994, service tax provisions are not applicable in Jammu and Kashmir. Service tax will not be payable if service is provided in Jammu & Kashmir, however, if a person from Jammu & Kashmir provides the service outside Jammu & Kashmir, the service will be liable to service tax and service tax shall be payable by the person who is receiving the service as per provisions of Reverse Charge. Since India includes Territorial Waters and also Exclusive Economic Zone, service tax shall be applicable in case of services provided in Indian Territorial Water and also Exclusive Economic Zone. Indian territorial waters extend upto 12 nautical miles from the Indian land mass. (1 nautical mile = 1.852 km) Exclusive Economic Zone of India extends upto 200 nautical miles from the baseline. Question 8: Write a note on charge of Service Tax. Answer: Charge of Service Tax Section 66B Section 66B is the charging section which deals with levy and collection of service tax. It prescribes the rate applicable for levying of service tax. The prescribed manner for collection and payment of service tax is provided in the Service Tax Rules, 1994. The rate of service tax at present is 12% and education cess and secondary and higher education cess is payable @ 3% on service tax. Service tax is payable on the value of taxable services and valuation is given in section 67 of the Finance Act, 1994.E.g. If a service provider has issued a bill of `10,00,000, service tax shall be `10,00,000 x 12.36% = `1,23,600 Question 9 (V. Imp.): What is the exemption limit for the Small Service Provider. Answer: As per Notification No.33/2012, dated 20.06.2012, exemption is allowed upto `10,00,000 provided in the preceding year, value of taxable services rendered is not exceeding `10,00,000. While computing amount of `10,00,000, value of services which are exempt from service tax shall not be considered. If any person is rendering services from more than one premises, value of services rendered from all the premises shall be taken into consideration. If any person is rendering more than one services, value of all the services shall be taken into consideration.

Service Tax

No CENVAT Credit is allowed till the service provider is availing exemption under Notification No. 33/2012. Example A Chartered Accountant has started his practice w.e.f 01.04.2012 and amount received upto 31.12.2012 is `10,00,000 and he has taken input services of `3,00,000 and paid service tax of `37,080, in this case, cenvat credit for input services is not allowed and also he cannot charge service tax on `10,00,000. If he has rejected general exemption of `10,00,000, in that case he should charge service tax of 10,00,000 x 12.36% = `1,23,600 and also cenvat credit is allowed for input services amounting to `37,080 and net tax payable shall be `86,520 The service provider has the option not to avail exemption and such option can not be withdrawn during the relevant year, however, it can be withdrawn from the beginning of next year. SELF READING [TO BE PUBLISHED IN THE GAZETTE OF INDIA, EXTRAORDINARY, PART II, SECTION 3, SUBSECTION (i)] Government of India Ministry of Finance (Department of Revenue) Notification No. 33/2012 - Service Tax New Delhi, the 20th June, 2012 G.S.R. (E).- In exercise of the powers conferred by sub-section (1) of section 93 of the Finance Act, 1994 (32 of 1994) (hereinafter referred to as the said Finance Act), and in supersession of the Government of India in the Ministry of Finance (Department of Revenue) notification No. 6/2005-Service Tax, dated the 1st March, 2005, published in the Gazette of India, Extraordinary, Part II, Section 3, Sub-section (i), vide G.S.R. number 140(E), dated the 1 st March, 2005, except as respects things done or omitted to be done before such supersession, the Central Government, being satisfied that it is necessary in the public interest so to do, hereby exempts taxable services of aggregate value not exceeding ten lakh rupees in any financial year from the whole of the service tax leviable thereon under section 66B of the said Finance Act: Provided that nothing contained in this notification shall apply to,(i) taxable services provided by a person under a brand name or trade name, whether registered or not, of another person; or (ii) such value of taxable services in respect of which service tax shall be paid by such person and in such manner as specified under sub-section (2) of section 68 of the said Finance Act read with Service Tax Rules,1994. 2. The exemption contained in this notification shall apply subject to the following conditions, namely:(i) the provider of taxable service has the option not to avail the exemption contained in this notification and pay service tax on the taxable services provided by him and such option, once exercised in a financial year, shall not be withdrawn during the remaining part of such financial year; (ii) the provider of taxable service shall not avail the CENVAT credit of service tax paid on any input services, under rule 3 or rule 13 of the CENVAT Credit Rules, 2004 (herein after referred to as the said

Service Tax

rules), used for providing the said taxable service, for which exemption from payment of service tax under this notification is availed of; (iii) the provider of taxable service shall not avail the CENVAT credit under rule 3 of the said rules, on capital goods received, during the period in which the service provider avails exemption from payment of service tax under this notification; (iv) the provider of taxable service shall avail the CENVAT credit only on such inputs or input services received, on or after the date on which the service provider starts paying service tax, and used for the provision of taxable services for which service tax is payable; (v) the provider of taxable service who starts availing exemption under this notification shall be required to pay an amount equivalent to the CENVAT credit taken by him, if any, in respect of such inputs lying in stock or in process on the date on which the provider of taxable service starts availing exemption under this notification; (vi) the balance of CENVAT credit lying unutilised in the account of the taxable service provider after deducting the amount referred to in sub-paragraph (v), if any, shall not be utilised in terms of provision under sub-rule (4) of rule 3 of the said rules and shall lapse on the day such service provider starts availing the exemption under this notification; (vii) where a taxable service provider provides one or more taxable services from one or more premises, the exemption under this notification shall apply to the aggregate value of all such taxable services and from all such premises and not separately for each premises or each services; and (viii) the aggregate value of taxable services rendered by a provider of taxable service from one or more premises, does not exceed ten lakh rupees in the preceding financial year. 3. For the purposes of determining aggregate value not exceeding ten lakh rupees, to avail exemption under this notification, in relation to taxable service provided by a goods transport agency, the payment received towards the gross amount charged by such goods transport agency under section 67 of the said Finance Act for which the person liable for paying service tax is as specified under sub-section (2) of section 68 of the said Finance Act read with Service Tax Rules, 1994, shall not be taken into account. Explanation.- For the purposes of this notification,(A) brand name or trade name means a brand name or a trade name, whether registered or not, that is to say, a name or a mark, such as symbol, monogram, logo, label, signature, or invented word or writing which is used in relation to such specified services for the purpose of indicating, or so as to indicate a connection in the course of trade between such specified services and some person using such name or mark with or without any indication of the identity of that person; (B) aggregate value means the sum total of value of taxable services charged in the first consecutive invoices issued during a financial year but does not include value charged in invoices issued towards such services which are exempt from whole of service tax leviable thereon under section 66B of the said Finance Act under any other notification. 4. This notification shall come into force on the 1st day of July, 2012. [F.No. 334 /01/2012- TRU] (Raj Kumar Digvijay) Under Secretary to the Government of India

Service Tax CERTAIN DEFINITIONS SECTION 65B OF FINANCE ACT 1994

10

(2) "advertisement" means any form of presentation for promotion of, or bringing awareness about, any event, idea, immovable property, person, service, goods or actionable claim through newspaper, television, radio or any other means but does not include any presentation made in person; (3) "agriculture" means the cultivation of plants and rearing of all life-forms of animals, except the rearing of horses, for food, fibre, fuel, raw material or other similar products; (4) "agricultural extension" means application of scientific research and knowledge to agricultural practices through farmer education or training; (5) "agricultural produce" means any produce of agriculture on which either no further processing is done or such processing is done as is usually done by a cultivator or producer which does not alter its essential characteristics but makes it marketable for primary market; (6) "Agricultural Produce Marketing Committee or Board" means any committee or board constituted under a State law for the time being in force for the purpose of regulating the marketing of agricultural produce; (9) "amusement facility" means a facility where fun or recreation is provided by means of rides, gaming devices or bowling alleys in amusement parks, amusement arcades, water parks, theme parks or such other places but does not include a place within such facility where other services are provided; (11) "approved vocational education course" means, (i) a course run by an industrial training institute or an industrial training centre affiliated to the National Council for Vocational Training offering courses in designated trades notified under the Apprentices Act, 1961 (52 of 1961); or (ii) a Modular Employable Skill Course, approved by the National Council of Vocational Training, run by a person registered with the Directorate General of Employment and Training, Union Ministry of Labour and Employment; or (iii) a course run by an institute affiliated to the National Skill Development Corporation set up by the Government of India; (12) "assessee" means a person liable to pay tax and includes his agent; (14) "authorised dealer of foreign exchange" shall have the meaning assigned to "authorised person" in clause (c) of section 2 of the Foreign Exchange Management Act, 1999 (42 of 1999); (15) "betting or gambling" means putting on stake something of value, particularly money, with consciousness of risk and hope of gain on the outcome of a game or a contest, whose result may be determined by chance or accident, or on the likelihood of anything occurring or not occurring; (17) "business entity" means any person ordinarily carrying out any activity relating to industry, commerce or any other business or profession; (18) "Central Electricity Authority" means the authority constituted under section 3 of the Electricity (Supply) Act, 1948 (54 of 1948); (19) "Central Transmission Utility" shall have the meaning assigned to it in clause (10) of section 2 of the Electricity Act, 2003 (36 of 2003); (20) "courier agency" means any person engaged in the door-to-door transportation of time-sensitive documents, goods or articles utilising the services of a person, either directly or indirectly, to carry or accompany such documents, goods or articles;

Service Tax

11

(23) "electricity transmission or distribution utility" means the Central Electricity Authority; a State Electricity Board; the Central Transmission Utility or a State Transmission Utility notified under the Electricity Act, 2003 (36 of 2003); or a distribution or transmission licensee under the said Act, or any other entity entrusted with such function by the Central Government or, as the case may be, the State Government; (24) "entertainment event" means an event or a performance which is intended to provide recreation, pastime, fun or enjoyment, by way of exhibition of cinematographic film, circus, concerts, sporting event, pageants, award functions, dance, musical or theatrical performances including drama, ballets or any such event or programme; (25) "goods" means every kind of movable property other than actionable claim and money; and includes securities, growing crops, grass, and things attached to or forming part of the land which are agreed to be severed before sale or under the contract of sale; (26) "goods transport agency" means any person who provides service in relation to transport of goods by road and issues consignment note, by whatever name called; (27) "India" means, (a) the territory of the Union as referred to in clauses (2) and (3) of article 1 of the Constitution; (b) its territorial waters, continental shelf, exclusive economic zone or any other maritime zone as defined in the Territorial Waters, Continental Shelf, Exclusive Economic Zone and other Maritime Zones Act, 1976 (80 of 1976); (c) the seabed and the subsoil underlying the territorial waters; (d) the air space above its territory and territorial waters; and (e) the installations, structures and vessels located in the continental shelf of India and the exclusive economic zone of India, for the purposes of prospecting or extraction or production of mineral oil and natural gas and supply thereof; (28) "information technology software" means any representation of instructions, data, sound or image, including source code and object code, recorded in a machine readable form, and capable of being manipulated or providing interactivity to a user, by means of a computer or an automatic data processing machine or any other device or equipment; (29) "inland waterway" means national waterways as defined in clause (h) of section 2 of the Inland Waterways Authority of India Act, 1985 (82 of 1985) or other waterway on any inland water, as defined in clause (b) of section 2 of the Inland Vessels Act, 1917 (1 of 1917); (30) "interest" means interest payable in any manner in respect of any moneys borrowed or debt incurred (including a deposit, claim or other similar right or obligation) but does not include any service fee or other charge in respect of the moneys borrowed or debt incurred or in respect of any credit facility which has not been utilised; (31) "local authority" means (a) a Panchayat as referred to in clause (d) of article 243 of the Constitution; (b) a Municipality as referred to in clause (e) of article 243P of the Constitution; (c) a Municipal Committee and a District Board, legally entitled to, or entrusted by the Government with, the control or management of a municipal or local fund; (d) a Cantonment Board as defined in section 3 of the Cantonments Act, 2006 (41 of 2006);

Service Tax (e) a regional council or a district council constituted under the Sixth Schedule to the Constitution; (f) a development board constituted under article 371 of the Constitution; or (g) a regional council constituted under article 371A of the Constitution;

12

(32) "metered cab" means any contract carriage on which an automatic device, of the type and make approved under the relevant rules by the State Transport Authority, is fitted which indicates reading of the fare chargeable at any moment and that is charged accordingly under the conditions of its permit issued under the Motor Vehicles Act, 1988 (59 of 1988) and the rules made thereunder; (33) "money" means legal tender, cheque, promissory note, bill of exchange, letter of credit, draft, pay order, traveller cheque, money order, postal or electronic remittance or any such similar instrument but shall not include any currency that is held for its numismatic value; (34) "negative list" means the services which are listed in section 66D; (35) "non-taxable territory" means the territory which is outside the taxable territory; (36) "notification" means notification published in the Official Gazette and the expressions "notify" and "notified" shall be construed accordingly; (37) "person" includes, (i) an individual, (ii) a Hindu undivided family, (iii) a company, (iv) a society, (v) a limited liability partnership, (vi) a firm, (vii) an association of persons or body of individuals, whether incorporated or not, (viii) Government, (ix) a local authority, or (x) every artificial juridical person, not falling within any of the preceding sub-clauses; (39) "prescribed" means prescribed by rules made under this Chapter; (40) "process amounting to manufacture or production of goods" means a process on which duties of excise are leviable under section 3 of the Central Excise Act, 1944 (1 of 1944) or any process amounting to manufacture of alcoholic liquors for human consumption, opium, Indian hemp and other narcotic drugs and narcotics on which duties of excise are leviable under any State Act for the time being in force; (41) "renting" means allowing, permitting or granting access, entry, occupation, use or any such facility, wholly or partly, in an immovable property, with or without the transfer of possession or control of the said immovable property and includes letting, leasing, licensing or other similar arrangements in respect of immovable property; (43) "securities" has the meaning assigned to it in clause (h) of section 2 of the Securities Contract (Regulation) Act, 1956 (42 of 1956);

Service Tax

13

(44) "service" means any activity carried out by a person for another for consideration, and includes a declared service, but shall not include (a) an activity which constitutes merely, (i) a transfer of title in goods or immovable property, by way of sale, gift or in any other manner; or (ii) such transfer, delivery or supply of any goods which is deemed to be a sale within the meaning of clause (29A) of article 366 of the Constitution; or (iii) a transaction in money or actionable claim; (b) a provision of service by an employee to the employer in the course of or in relation to his employment; (c) fees taken in any Court or tribunal established under any law for the time being in force. Explanation 1. For the removal of doubts, it is hereby declared that nothing contained in this clause shall apply to, (A) the functions performed by the Members of Parliament, Members of State Legislative, Members of Panchayats, Members of Municipalities and Members of other local authorities who receive any consideration in performing the functions of that office as such member; or (B) the duties performed by any person who holds any post in pursuance of the provisions of the Constitution in that capacity; or (C) the duties performed by any person as a Chairperson or a Member or a Director in a body established by the Central Government or State Governments or local authority and who is not deemed as an employee before the commencement of this section. Explanation 2. For the purposes of this clause, transaction in money shall not include any activity relating to the use of money or its conversion by cash or by any other mode, from one form, currency or denomination, to another form, currency or denomination for which a separate consideration is charged. Explanation 3. For the purposes of this Chapter, (a) an unincorporated association or a body of persons, as the case may be, and a member thereof shall be treated as distinct persons; (b) an establishment of a person in the taxable territory and any of his other establishment in a nontaxable territory shall be treated as establishments of distinct persons. Explanation 4. A person carrying on a business through a branch or agency or representational office in any territory shall be treated as having an establishment in that territory; (46) "stage carriage" shall have the meaning assigned to it in clause (40) of section 2 of the Motor Vehicles Act, 1988 (59 of 1988); (47) "State Electricity Board" means the Board constituted under section 5 of the Electricity (Supply) Act, 1948 (54 of 1948); (48) "State Transmission Utility" shall have the meaning assigned to it in clause (67) of section 2 of the Electricity Act, 2003 (36 of 2003);

Service Tax

14

(49) "support services" means infrastructural, operational, administrative, logistic, marketing or any other support of any kind comprising functions that entities carry out in ordinary course of operations themselves but may obtain as services by outsourcing from others for any reason whatsoever and shall include advertisement and promotion, construction or works contract, renting of immovable property, security, testing and analysis; (50) "tax" means service tax leviable under the provisions of this Chapter; (51) "taxable service" means any service on which service tax is leviable under section 66B; (52) "taxable territory" means the territory to which the provisions of this Chapter apply; (54) "works contract" means a contract wherein transfer of property in goods involved in the execution of such contract is leviable to tax as sale of goods and such contract is for the purpose of carrying out construction, erection, commissioning, installation, completion, fitting out, repair, maintenance, renovation, alteration of any movable or immovable property or for carrying out any other similar activity or a part thereof in relation to such property; (55) words and expressions used but not defined in this Chapter and defined in the Central Excise Act, 1944 (1 of 1944) or the rules made thereunder, shall apply, so far as may be, in relation to service tax as they apply in relation to a duty of excise. (Applicable from 01.07.2012) Question 10: Write a note on services covered in the Negative List. Answer: Negative list of services Section 66D The negative list shall comprise of the following services, namely: (a) services by Government or a local authority except support services. As per section 65B(49), "Support services" means infrastructural, operational, administrative, logistic, marketing or any other support of any kind comprising functions that entities carry out in ordinary course of operations themselves but may obtain as services by outsourcing from others for any reason whatsoever and shall include advertisement and promotion, construction or works contract, renting of immovable property, security, testing and analysis. (b) services by the Reserve Bank of India; (c) services by a foreign diplomatic mission located in India (as per notification no 27/2012 dated: 20.06.2012, services provided to foreign diplomatic mission shall also be exempt from service tax and even services provided for personal use of the employees or their family members shall also be exempt from service tax); (d) services relating to agriculture by way of (i) agricultural operations directly related to production of any agricultural produce including cultivation, harvesting, threshing, plant protection or seed testing; (ii) supply of farm labour; (iii) processes carried out at an agricultural farm including tending, pruning, cutting, harvesting, drying, cleaning, trimming, sun drying, fumigating, curing, sorting, grading, cooling or bulk

Service Tax

15

packaging and such like operations which do not alter the essential characteristics of agricultural produce but make it only marketable for the primary market; (iv) renting or leasing of agro machinery or vacant land with or without a structure incidental to its use; (v) loading, unloading, packing, storage or warehousing of agricultural produce; (vi) agricultural extension services; (vii) services by any Agricultural Produce Marketing Committee or Board or services provided by a commission agent for sale or purchase of agricultural produce; As per section 65B(3), "Agriculture" means the cultivation of plants and rearing of all lifeforms of animals, except the rearing of horses, for food, fibre, fuel, raw material or other similar products. As per section 65B(4), "Agricultural extension" means application of scientific research and knowledge to agricultural practices through farmer education or training. As per section 65B(5), "Agricultural produce" means any produce of agriculture on which either no further processing is done or such processing is done as is usually done by a cultivator or producer which does not alter its essential characteristics but makes it marketable for primary market. (e) trading of goods; (f) any process amounting to manufacture or production of goods; As per section 65B(40), "Process amounting to manufacture or production of goods" means a process on which duties of excise are leviable under section 3 of the Central Excise Act, 1944 (1 of 1944) or any process amounting to manufacture of alcoholic liquors for human consumption, opium, Indian hemp and other narcotic drugs and narcotics on which duties of excise are leviable under any State Act for the time being in force. (g) selling of space or time slots for advertisements other than advertisements broadcast by radio or television; (h) service by way of access to a road or a bridge on payment of toll charges; (i) betting, gambling or lottery; As per section 65B(15), "Betting or gambling" means putting on stake something of value, particularly money, with consciousness of risk and hope of gain on the outcome of a game or a contest, whose result may be determined by chance or accident, or on the likelihood of anything occurring or not occurring. (j) admission to entertainment events or access to amusement facilities;

Service Tax

16

As per section 65B(9), "Amusement facility" means a facility where fun or recreation is provided by means of rides, gaming devices or bowling alleys in amusement parks, amusement arcades, water parks, theme parks or such other places but does not include a place within such facility where other services are provided; As per section 65B(24), "Entertainment event" means an event or a performance which is intended to provide recreation, pastime, fun or enjoyment, by way of exhibition of cinematographic film, circus, concerts, sporting event, pageants, award functions, dance, musical or theatrical performances including drama, ballets or any such event or programme; (k) transmission or distribution of electricity by an electricity transmission or distribution utility; As per section 65B(23), "Electricity transmission or distribution utility" means the Central Electricity Authority; a State Electricity Board; the Central Transmission Utility or a State Transmission Utility notified under the Electricity Act, 2003 (36 of 2003); or a distribution or transmission licensee under the said Act, or any other entity entrusted with such function by the Central Government or, as the case may be, the State Government. (l) services by way of (i) pre-school education and education up to higher secondary school or equivalent; (ii) education as a part of a curriculum for obtaining a qualification recognised by any law for the time being in force; (iii) education as a part of an approved vocational education course; As per section 65B(11), "Approved vocational education course" means, (i) a course run by an industrial training institute or an industrial training centre affiliated to the National Council for Vocational Training offering courses in designated trades notified under the Apprentices Act, 1961 (52 of 1961); or (ii) a Modular Employable Skill Course, approved by the National Council of Vocational Training, run by a person registered with the Directorate General of Employment and Training, Union Ministry of Labour and Employment; or (iii) a course run by an institute affiliated to the National Skill Development Corporation set up by the Government of India; (m) services by way of renting of residential dwelling for use as residence; (n) services by way of (i) extending deposits, loans or advances in so far as the consideration is represented by way of interest or discount; (ii) inter se sale or purchase of foreign currency amongst banks or authorised dealers of foreign exchange or amongst banks and such dealers; (o) service of transportation of passengers, with or without accompanied belongings, by

Service Tax (i) a stage carriage; (ii) railways in a class other than (A) first class; or (B) an airconditioned coach; (iii) metro, monorail or tramway; (iv) inland waterways;

17

(v) public transport, other than predominantly for tourism purpose, in a vessel of less than fifteen tonne net; and (vi) metered cabs, radio taxis or auto rickshaws; (p) services by way of transportation of goods (i) by road except the services of (A) a goods transportation agency; or (B) a courier agency; (ii) by an aircraft or a vessel from a place outside India to the first customs station of landing in India; or (iii) by inland waterways; (q) funeral, burial, crematorium or mortuary services including transportation of the deceased. (Applicable from 01.07.2012) Principles of interpretation of specified descriptions of services or bundled services Section 66F (1) Unless otherwise specified, reference to a service (herein referred to as main service) shall not include reference to a service which is used for providing main service. (2) Where a service is capable of differential treatment for any purpose based on its description, the most specific description shall be preferred over a more general description. (3) Subject to the provisions of sub-section (2), the taxability of a bundled service shall be determined in the following manner, namely: (a) if various elements of such service are naturally bundled in the ordinary course of business, it shall be treated as provision of the single service which gives such bundle its essential character; (b) if various elements of such service are not naturally bundled in the ordinary course of business, it shall be treated as provision of the single service which results in highest liability of service tax.

Service Tax

18

Explanation. For the purposes of sub-section ( 3), the expression "bundled service" means a bundle of provision of various services wherein an element of provision of one service is combined with an element or elements of provision of any other service or services. Applicable from 01.07.2012 Question 11 (V. Imp.): Write a note on Registration under Service Tax. Answer: Registration Section 69/Rule 4(1) of STR, 1994 A service provider can charge service tax only if he is registered with the Service Tax Department. Every service provider in whose case, gross receipt has exceeded `9,00,000 shall apply for registration to the Service Tax Department in Form No ST-1 within 30 days from the date of crossing the limit of `9,00,000, however, he will charge service tax after crossing the limit of `10,00,000. He will also submit a copy of Permanent Account Number and proof of residence. Department shall grant him a registration certificate in Form No ST-2 within 7 days from the date of receipt of application otherwise the service provider shall be deemed to be registered. The service provider shall be given a Registration Number by the Department which will be called STP code i.e. Service Tax Payer Code and it will be 15 digit PAN based number and first 10 digit shall be that of PAN and remaining 5 digit shall be allotted by Service Tax Department e.g. AAEPC1298D ST-001 The last three digit shall indicate total number of registration for the same Permanent Account Number. If a person is providing more than one taxable service, he may give only one application. He should mention in the application all the taxable services provided by him. A person may apply for voluntary registration at any time and also a person may forgo (reject) the general exemption of `10,00,000. As per section 77, any person who has failed to take registration shall pay penalty of `200 per day or `10,000 whichever is higher. Question 12 (V. Imp.): Write a note on Centralized Registration under Service Tax. Answer: Centralised Registration Rule 4(2) of STR, 1994 If a service provider is providing services from more than one premises, in such cases he can apply for separate registration for each of such premises. He may apply for a single registration called Centralized Registration provided he has either centralized accounting or centralized billing system. In such cases one of the places shall be considered to be Head Office and all other premises shall be considered to be branches. A single registration certificate shall be issued. If the service provider do not have centralized accounting and also there is no centralized billing system, he will not be allowed to apply for centralized registration. Question 13: Write a note on Amendment of Registration Certificate. Answer: Amendment of Registration Certificate Rule 4(5A) of STR, 1994 A service provider may apply for amendment in registration in the following cases: 1. Change in place of business 2. Change in the name of business 3. Change in services rendered i.e. there may be addition / deletion 4. Any other similar change If there is any such change, service provider should apply to the department within 30 days for effecting the change.

Service Tax

19

Question 14: Write a note on Cancellation of Registration Certificate. Answer: Cancellation of Registration Certificate Rule 4(7), 4(8) of STR, 1994 Every registered assessee, who ceases to provide the taxable service for which he is registered or if he dies, registration certificate shall be cancelled. The Superintendent of Central Excise shall ensure that the assessee has paid all monies due to the Central Government under the provisions of the Act, and the rules and the notifications issued thereunder, and thereupon cancel the registration certificate. Where a registered assessee transfers his business to another person, the transferee shall obtain a fresh certificate of registration. Question 15: Write a note on issue of Invoice/Bill by Service Provider. Answer: As per Rule 4A of STR, 1994, every person providing taxable service, not later than 30 days from the date of completion of such taxable service or receipt of any payment towards the value of such taxable service, whichever is earlier, shall issue an invoice, a bill or, signed by such person or a person authorized by him and such challan etc. shall be serially numbered and shall contain the following, namely: (i) the name, address and the registration number of such person; (ii) the name and address of the person receiving taxable service; (iii) description and value of taxable service provided or agreed to be provided; and (iv) the service tax payable thereon : Provided also that in case of continuous supply of service, every person providing such taxable service shall issue an invoice, bill or challan, as the case may be, within 30 days of the date when each event specified in the contract, which requires the service receiver to make any payment to service provider, is completed: Every input service distributor distributing credit of taxable services shall, in respect of credit distributed, issue an invoice, a bill or, as the case may be, a challan signed by such person or a person authorized by him, for each of the recipient of the credit distributed, and such invoice, bill or, as the case may be, challan shall be serially numbered and shall contain the following, namely: (i) the name, address and registration number of the person providing input services and the serial number and date of invoice, bill, or as the case may be; (ii) the name and address of the said input service distributor; (iii) the name and address of the recipient of the credit distributed; (iv) the amount of the credit distributed : Question 16 (V. Imp): Write a note on payment of Service Tax. Answer: Payment of Service Tax Section 68/ Rule 6 of STR, 1994 An individual or proprietary firm or partnership firm shall be required to pay service tax on quarterly basis and should be paid upto 5th of the month subsequent to the quarter in which the service is deemed to be provided (i.e. the quarter in which point of taxation falls) but if the payment is being made electronically through internet banking, payment can be made upto 6 th of such month. However, payment for the last quarter should be made upto 31st March of that year.

Service Tax

20

Any other service provider like Company or HUF etc. shall be required to pay service tax on monthly basis and it should be paid upto 5th of the month succeeding the month in which service is deemed to be provided but if payment is being made electronically through internet banking, payment can be made upto 6 th of the month however, payment for last month should be made upto 31st March of that year If an assessee has paid a total service tax of rupees ten lakh or more including the amount paid by utilization of Cenvat Credit, in the preceding financial year, he shall deposit the service tax liable to be paid by him electronically, through internet banking. As per section 77, if any person has failed to pay service tax electronically, he shall pay penalty which may extend upto `10,000. The assessee shall deposit the service tax liable to be paid by him in Form G.A.R-7 with the bank designated by the Central Board of Excise and Customs for this purpose. If the assessee deposits the service tax by cheque, the date of presentation of cheque to the bank designated by the Central Board of Excise and Customs for this purpose shall be deemed to be the date on which service tax has been paid subject to realization of that cheque. As per Rule 6 of STR 1994, all service provider who are individual or partnership firm shall pay service tax on actual receipt basis upto a value of `50,00,000 and on the excess over it on the basis of Rule 3 of POT. Further aggregate value of service rendered in the immediately preceding year should not exceed `50,00,000. Example In the financial year 2012-13, Mr. Y a Chartered Accountant has rendered services having a value of 150 lakhs, (value of services rendered in Financial Year 2011-12 is `50 lakhs) in this case, as per Rule 6 of STR 1994, CA should pay service tax with regard to first 50 Lakh on actual receipt basis and on balance service tax should be paid as per rule 3 of POT Rules 2011. If value of services rendered in Financial Year 2011-12 is more than `50 lakhs, in Financial Year 2012-13, CA should pay service tax as per Rule 3 of POT Rules 2011 i.e. payment of service tax shall not be on actual receipt basis. Example Mr. X is a service provider and value of service rendered during the year as follows: F.Y. 2011-12 `45 Lakh F.Y. 2012-13 `130 Lakh In this case in F.Y. 2012-13, service tax shall be paid on actual receipt basis on first ` 50 Lakh and on balance amount of ` 80 Lakh service tax shall be paid as per Rule 3 of Point of Taxation Rules 2011. If in this case service provider is a company, service tax shall be payable on the entire value of service as per the Rule 3 of Point of Taxation Rules 2011. If service provider is individual and value of the service rendered in F.Y.2011-12 is ` 65 Lakh, service tax shall be payable as per the Rule 3 of Point of Taxation Rule 2011. Rounding off of Duty, etc Section 37D (Central Excise Act, 1944) The amount of duty, interest, penalty, fine or any other sum payable, and the amount of refund or any other sum due, under the provisions of this Act shall be rounded off to the nearest rupee and, for this purpose, where such amount contains a part of a rupee consisting of paise then, if such part is fifty paise or more, it shall be increased to one rupee and if such part is less than fifty paise it shall be ignored.

Service Tax

21

Question 17 (V. Imp.): Write a note on Point of Taxation for Payment of Service Tax. Answer: Point of Taxation for Payment of Service Tax Rule 3 Prior to 01.04.2011, service tax was to be paid on actual receipt basis but w.e.f. 01.04.2011, service tax shall be paid on the basis of point of taxation i.e. the point in time when the service shall be deemed to have been provided shall be taken into consideration. As per Rule 3, services shall be deemed to have been provided on the date of invoice issued by the service provider but if the invoice has been issued after expiry of 30 days from the date of completion of service, services shall be deemed to have been provided on the date of completion of service. If payment has been received before the date determined above, the date of providing the services shall be the date of receiving the payment and service tax should be paid accordingly. It can be further shown as given below: S. No. 1. 2. 3. 4. Date of completion of service April 10, 2012 April 10, 2012 April 10, 2012 April 10, 2012 April 20, 2012 May 26, 2012 April 20, 2012 May 26, 2012 Date of invoice Date on which payment recd. April 30, 2012 May 31, 2012 April 15, 2012 April 5, 2012 (part payment) and May 25, 2012 (balance payment) 15th April 2012 Point of Taxation April 20, 2012 April 10, 2012 April 15, 2012 April 5, 2012 (part payment) and April 10, 2012 (balance payment) 01st April 2012 Invoice issued in 30 days and before receipt of payment Invoice not issued within 30 days and payment received after completion of service Invoice issued in 30 days but payment received before invoice Invoice not issued in 30 days, point of taxation 10th April but for part payment received on 05th April 2012, POT shall be 05th April 2012. Invoice issued before completion of service Remarks

5.

April 10, 2012

1st April 2012

As per Rule 3 of POT Rules 2011, wherever the provider of taxable service receives a payment up to rupees one thousand in excess of the amount indicated in the invoice. The service provider shall have the option not to issue separate invoice for such excess amount and date of receiving payment of such excess amount shall not be the POT for such excess amount rather the date of issue of invoice for rendering services shall be the POT. E.g. If Airtel Limited has issued a bill of `950 on 05.05.2012 to Mr. X for the services rendered in April 2012 and Mr. X deposited `1,000, in this case the company has option not to issue separate invoice for excess amount of `50 and POT for `50 shall not be 05.05.2012. Company issued bill of `850 on 05.06.2012 to Mr. X for services rendered in May 2012, in this case excess amount of `50 shall be adjusted in bill of May 2012 and POT shall be 05.06.2012. Continuous Supply of service means any service which is provided, or agreed to be provided continuously or on recurrent basis, under a contract, for a period exceeding three months with the obligation for payment periodically or from time to time, or where the Central Government, by a notification in the Official Gazette, prescribes provision of a particular service to be a Continuous Supply of service

Service Tax

22

Illustration 1: ABC Ltd. is a construction company and it has entered into an agreement with XYZ Ltd. to construct one building which will have 4th floor and as per the contract, each floor shall be considered to be one stage of completion. ABC Ltd. submitted particulars as given below: Completed Ground Floor Issued Bill Half payment of ` 10,00,000 plus service tax received And balance payment of ` 10,00,000 plus Service Tax received Completed 1st floor Issued Bill Half payment of ` 9,00,000 plus service tax received And balance payment of ` 9,00,000 plus Service Tax received Completed 2nd Floor Issued Bill Half payment of ` 8,00,000 plus service tax received And balance payment of ` 8,00,000 plus Service Tax received Completed 3rd Floor Issued Bill Full amount of ` 10,00,000 plus Service Tax received on 16/06/2012 on 01/07/2012 on 27/06/2012 on 10/07/2012 on 27/08/2012 on 07/09/2012 on 31/08/2012 on 10/09/2012 on 30/11/2012 on 10/11/2012 on 30/10/2012 on 01/12/2012 on 10/03/2013 on 01/04/2013 on 01/08/2013

In FY 2011-12, Bill issued by company were ` 200 lakh and service tax was paid @ 12.36% Show tax treatment of service tax for each month. Solution: April 2012 May 2012 June 2012 Taxable value Service tax @ 12.36% July 2012 Taxable value Service tax @ 12.36% Aug 2012 Taxable value Service tax @ 12.36% Sept 2012 Taxable value Service tax @ 12.36% ` NIL NIL 10,00,000 1,23,600 10,00,000 1,23,600 9,00,000 1,11,240 9,00,000 1,11,240

Service Tax Oct 2012 Taxable value Service tax @ 12.36% Nov 2012 Taxable value Service tax @ 12.36% Dec 2012 Jan, Feb 2013 March 2013

23 8,00,000 98,880 8,00,000 98,880 NIL NIL NIL

Question 18: Write a note on interest on delayed payment of Service Tax. Answer: Interest on delayed payment of service tax Section 75 If any person has failed to credit the tax to the Central Government within the prescribed time period, such person shall pay interest for the period of delay at the prescribed rate. As per notification no. 14/2011 dated 01.03.2011, the rate of interest shall be 18% p.a. Provided further that in the case of a service provider, whose value of taxable services provided in the preceding financial year is upto `60 lakhs, rate of interest shall be 15% p.a. Question 19: Write a note on penalty for failure to pay Service Tax. Answer: Penalty for failure to pay service tax Section 76 If any person liable to pay service tax has failed to pay service tax, he shall be liable to pay service tax alongwith interest @ 18% p.a. and also penalty may be imposed which shall be `100 per day or 1% of such tax, per month whichever is higher starting with first day after the due date till the date of actual payment. However total amount of penalty shall not exceed 50% of service tax payable. Example ABC Ltd, an assessee, fails to pay service tax of ten lakh rupees payable by 5 th March. ABC Ltd. pays amount on the 15th March. The default has continued for ten days The penalty payable by ABC Ltd. is computed as follows: 100 x 10 = `1,000 10,00,000 x 1% x 10/31 = `3,225.80 whichever is higher Penalty liable to be paid is ` 3226.00. Example Mr. X, an assessee, fails to pay service tax of `3,000 payable by 31st March. Mr. X pays the amount on 30 th April. The default has continued for 30 days. The penalty payable by Mr. X is computed as follows: 100 x 30 = `3,000 3,000 x 1% x 30/30 = `30 whichever is higher. Penalty liable to be paid is `3,000 but penalty cannot exceed 50% of the amount of service tax, hence amount of penalty shall be `1,500

Service Tax

24

Question 20: Write a note on penalty for contravention of rules and provisions of Act for which no penalty is specified elsewhere. Answer: Penalty for contravention of rules and provisions of Act for which no penalty is specified elsewhere Section 77 (1) Any person who has failed to take registration shall pay penalty of `200 per day or `10,000 whichever is higher (2) Any person who has failed to maintain the book of accounts shall pay penalty which may extend upto `10,000. (3) Any person who has failed to furnish information or failed to furnish any document to the service tax officer shall pay penalty of `200 per day or `10,000 whichever is higher. (4) If any person has failed to pay service tax electronically shall pay penalty which may extend upto `10,000 (5) Any person who issues invoice with incorrect or incomplete details or fails to account for an invoice in his books of account, shall be liable to a penalty which may extend to ten thousand rupees. (6) Any person, who contravenes any of the provisions of this Chapter or any rules made thereunder for which no penalty is separately provided in this Chapter, shall be liable to a penalty which may extend to ten thousand rupees. Question 21: Write a note on penalty for suppressing (to conceal) value of taxable services. Answer: Penalty for suppressing value of taxable service Section 78 If any person has not paid service tax or has short paid service tax or there was a refund of service tax because of fraud, wilful mis-statement etc., in such cases he shall be liable to pay service tax alongwith interest and also penalty shall be payable which shall be equal to the amount of service tax. Provided that where true and complete details of the transactions are available in the specified records, penalty shall be reduced to 50% of the service tax so not levied or paid or short-levied or short-paid or erroneously (wrongly) refunded. If the tax and interest has been paid within 30 days of receiving the orders from the Central Excise Officer, penalty payable shall be 25% of such service tax. Further such penalty should also be paid within the period of 30 days as mentioned above. If the service provider whose value of taxable services do not exceeds `60 lakhs during preceding financial year, the period of 30 days shall be taken as 90 days. Provided also that if the penalty is payable under this section, the provisions of section 76 shall not apply. Question 22 (Imp.): Write a note on Advance Payment of Service Tax. Answer: Advance payment of Service Tax Rule 6 of STR, 1994 Every person liable to pay service tax, may, on his own volition (voluntary), pay an amount as service tax in advance, to the credit of the Central Government and adjust the amount so paid against the service tax which he is liable to pay for the subsequent period :

Service Tax

25

Provided that the assessee shall, (i) intimate the details of the amount of service tax paid in advance, to the jurisdictional Superintendent of Central Excise within a period of fifteen days from the date of such payment; and (ii) indicate the details of the advance payment made, and its adjustment, if any in the subsequent return to be filed under section 70 of the Act. Question 23 (Imp.): Write a note on Refund of Service Tax/ Issue of Credit Note. Answer: Refund of Service Tax / Issue of Credit Note Rule 6 of STR, 1994 Where an assessee has issued an invoice, or received any payment, against a service to be provided which is not so provided by him either wholly or partially for any reason or where the amount of invoice is renegotiated due to deficient provision of service, or any terms contained in a contract, the assessee may take the credit of such excess service tax paid by him, if the assessee, (a) has refunded the payment or part thereof, so received for the service provided to the person from whom it was received; or (b) has issued a credit note for the value of the service not so provided to the person to whom such an invoice had been issued. Question 24 (Imp.): Write a note on Provisional Payment of Service Tax. Answer: Provisional payment of Service Tax Rule 6 of STR, 1994 In case the assessee is unable to correctly estimate, at the time of the deposit, the actual amount of service tax for any month or quarter, he may make a written request to Assistant/ Deputy Commissioner of Central Excise for making payment of service tax on provisional basis. The concerned officer may allow payment of service tax on provisional basis on such value of taxable service as may be specified by him. For the purpose of provisional assessment at the time of filing the return, the assessee is required to file a statement in form ST - 3A giving detail of difference between service tax deposited and the service tax liable to be paid for each month. The Assistant/Deputy Commissioner of Central Excise, on the basis of memorandum in form ST - 3A may complete the assessment after calling for necessary documents or records, if need be. Question 25 (V. Imp.): Write a note on Self Adjustment of Service Tax. Answer: Self adjustment of Service Tax Rule 6 of STR, 1994 If an assessee has paid to the credit of Central Government any amount in excess of the amount required to be paid towards service tax liability for a month or quarter, as the case may be, the assessee may adjust such excess amount paid by him against his service tax liability for the succeeding month or quarter, as the case may be. The adjustment of excess amount paid, shall be subject to the condition that the excess amount paid is on account of reasons not involving interpretation of law, taxability, valuation or applicability of any exemption notification. The details and reasons for such adjustment shall be intimated to the jurisdictional Superintendent of Central Excise within a period of fifteen days from the date of such adjustment. As per section 11B of Central Excise Act, 1944 if excess amount paid is on account of reasons involving interpretation of law, taxability, valuation or applicability of any exemption notification, in such cases, the service provider should claim refund and for this purpose he should make an application before expiry of one year from the date of making the payment to Central Excise Officer and he shall grant him refund

Service Tax

26

provided incidence of service tax has not been passed on to some other person otherwise amount shall be credited to Consumer Welfare Fund. As per section 11BB if refund has not been granted within 3 months from the date of receipt of the application, interest shall be allowed @ 6% p.a. from the date immediately after expiry of the period of 3 months till the date of refund (Notification No. 67/2003 dated: 12.09.2003) Question 26: Write a note on adjustment for Municipal Tax paid for House Property. Answer: Adjustment for Municipal Tax paid for house property Rule 6 of STR, 1994 Where the person liable to pay service tax in respect of renting of immovable property, has paid any amount in excess of the amount required to be paid towards service tax liability on account of non-availment of deduction of property tax paid, from the gross amount charged for renting of the immovable property, the assessee may adjust such excess amount paid by him against his service tax liability within one year from the date of payment of such property tax. The details of such adjustment shall be intimated to the Superintendent of Central Excise within a period of fifteen days from the date of such adjustment. Example X Ltd. Gives a commercial property on rent to Y Ltd. (monthly rent being `2,00,000) Y Ltd. Pays rent on the last day of each month. Municipal tax for the financial year 2012-13 is `4,44,000 (due date of payment is June 30, 2012, whereas it is actually paid on September 30, 2012 along with late payment interest of `16,200) Value of taxable service for the financial Year 2012-13 will be as follows: Financial Year 2012-13 April May June July August September October November December January February March Gross rent ` 2,00,000 2,00,000 2,00,000 2,00,000 2,00,000 2,00,000 2,00,000 2,00,000 2,00,000 2,00,000 2,00,000 2,00,000 Municipal tax (`4,44,000 12 = `37,000) ` 1,85,000 + 37,000 = 2,22,000 37,000 + 22,000 = 59,000 37,000 37,000 37,000 37,000 37,000 Taxable Value of service ` 2,00,000 2,00,000 2,00,000 2,00,000 2,00,000 (-) 22,000 1,41,000 1,63,000 1,63,000 1,63,000 1,63,000 1,63,000

Municipal tax is paid on September 30, 2012. Municipal tax liability from April 2012 till August 2012 comes to `1,85,000 (i.e., `37,000 5). This unadjusted amount can be adjusted at any time within one year from the date of payment of municipal tax. The details of such adjustment should be informed to jurisdictional Superintendent of Central Excise within period of 15 days from date of adjustment [rule 6(4C) of Service Tax Rules] Question 27 (Imp.): Write a note on service tax collected wrongly or in excess from any person.

Service Tax

27

Answer: As per section 73A, if any person has collected any service tax in excess or wrongly from the recipient of taxable service in any manner, in that case such person should deposit such amount with the Central Government. Central Government shall refund the amount to the person from whom tax has been collected and for this purpose Central Government may issue a public notice and such person can make an application within 6 months from the date of public notice. If the amount has not been claimed by any person, it will be deposited in the Consumer Welfare Fund which is regulated by a committee and its members are nominated by the Government. The amount is utilized for the welfare of consumer in India. As per section 73B, such person shall also pay interest @ 18% p.a. from the first day of the month succeeding the month in which the amount ought to have been paid, till the date of payment of such amount. If the total value of taxable services in the preceding year is upto `60,00,000, rate of interest shall be 15% p.a. Question 28 (Imp.): Write a note on Point of Taxation for determining the rate of Service Tax. Answer: Point of taxation for determining the rate of Service Tax Rule 4 of POT Rules, 2011 Rate of service tax shall be determined as per Rule 4 in the manner given below: Case 1: Taxable services have been completed before the change in effective rate of tax Situation where the invoice issued as well as the payment received after the change in effective rate of tax where the invoice is issued prior to change in effective rate of tax but the payment is received after the change in effective rate of tax where the payment is received before the change in effective rate of tax, but the invoice for the same has been issued after the change in effective rate of tax It will be further clear with help of the following illustrations. S. No. 1. 2. 3. Date of completion of service April 10, 2012 April 10, 2012 April 10, 2012 Change in effective rate of tax April 25, 2012 April 25, 2012 April 25, 2012 April 26, 2012 April 15, 2012 April 30, 2012 April 30, 2012 April 30, 2012 April 15, 2012 April 26, 2012 April 15, 2012 April 15, 2012 New rate Old rate Old rate Date of invoice Date on which payment recd. Point of Taxation Rate applicable Date of Receipt Point of Taxation Date of Receipt or issuing of invoice, whichever is earlier Date of Issue of Invoice

Case 2: Services have been completed after the change in effective rate of tax

Service Tax Situation change in effective rate of tax but the invoice has been Point of Taxation

28

where the payment for the invoice is made after the date of Receipt of payment issued prior to the change in effective rate of tax where the invoice has been issued and the payment date of receipt of payment or date of issuance of for the invoice received before the change in effective invoice, whichever is earlier rate of tax where the invoice has also been raised after the date of issuing of invoice change in effective rate of tax but the payment has been received before the change in effective rate of tax The applicability of the rule will be clear from the illustrations in the following table: S. No. 1. 2. 3. Date of completion of service April 30, 2012 April 30, 2012 April 30, 2012 Change in effective rate of tax April 25, 2012 April 25, 2012 April 25, 2012 Date of invoice April 20, 2012 April 20, 2012 April 30, 2012 Date on which payment recd. April 30, 2012 April 15, 2012 April 20, 2012 Point of Taxation April 30, 2012 April 15, 2012 April 30, 2012 Rate of Tax New rate Old rate New rate

Question 29: Write a note on liability to service tax in case of New Services. Answer: New Services Rule 5 of POT Rules, 2011 Where a service is taxed for the first time, then, (a) no tax shall be payable to the extent the invoice has been issued and the payment received against such invoice before such service became taxable; (b) no tax shall be payable if the payment has been received before the service becomes taxable and invoice has been issued within fourteen days of the date when the service is taxed for the first time. Question 30: Explain the meaning of the date of receiving the payment. Answer: As per Rule 2A of POT Rules 2011, date of payment shall be the earlier of the dates on which the payment is entered in the books of accounts or is credited to the bank account of the person liable to pay tax: e.g. ABC Ltd. has received a bank draft of ` 5,00,000 towards the services rendered and payment was entered in the books of accounts on 1st July 2012, and draft was credited in the bank account on 7 th July 2012, in this case date of payment shall be 1st July, 2012, but if amount was credited in bank account on 7th July 2012 but payment was entered in the books of accounts on 10th July 2012, in this case date of payment shall be 7th July 2012. However, the date of payment shall be the date of credit in the bank account when there is a change in effective rate of tax or when a service is taxed for the first time during the period between such entry in books of accounts and its credit in the bank account and the credit in the bank account is after four working days from the date when there is change in effective rate of tax or a service is taxed for the first time. E.g. Services are rendered on 01.07.2012 and payment is entered in the books of accounts on 03.07.2012 and there is change in rate of service tax on 04.07.2012 and amount is credited to the bank account through bank draft on 06.07.2012, in this case date of payment shall be 03.07.2012 and old rate shall be applicable

Service Tax

29

but if payment is credited to the bank account 09.07.2012, date of payment shall be the date of credit in the bank account and accordingly new rate shall be applicable. Question 31: Write a note on determination of Point of Taxation in case of Copyrights, etc. (For Self Reading) Answer: Determination of Point of Taxation in case of Copyrights, etc. Rule 8 In respect of royalties and payments pertaining to copyrights, trademarks, designs or patents, where the whole amount of the consideration for the provision of service is not ascertainable at the time when service was performed, and subsequently the use or the benefit of these services by a person other than the provider gives rise to any payment of consideration, the service shall be treated as having been provided each time when a payment in respect of such use or the benefit is received by the provider in respect thereof, or an invoice is issued by the provider, whichever is earlier. Question 32(V. Imp.): Write a note on Filing of Service Tax Return. Answer: Furnishing of Returns Section 70 / Rule 7 of STR, 1994 Every assessee liable to pay service tax shall submit a half yearly return in From ST-3 or ST-3A, as the case may be, along with a copy of the Form GAR-7 for the months / quarters covered in the half-yearly return. Every assessee shall submit the half yearly return by the 25th of the month following the particular half-year. If the last date is a holiday, return can be filed on the next day without any penalty. Every assessee shall submit the half-yearly return electronically. Every service provider who is registered shall also be required to file the return even if service tax liability is nil i.e. if any service provider has gross receipt exceeding `9,00,000 shall be required to get registered and in that case return has to be filed even if service tax liability is nil. Every assessee shall furnish to the Superintendent of Central Excise at the time of filing his return for the first time a list of all accounts maintained by the assessee in relation to service tax including memoranda received from his branch offices. Question 33: Write a note on Revision of Service Tax Return. Answer: Revision of Return Rule 7B of STR, 1994 An assessee may submit a revised return, in Form ST-3, in triplicate, to correct a mistake or omission, within a period of 90 days from the date of submission of the return under rule 7. Question 34 (V. Imp.): Write a note on Delay Furnishing of Service Tax Return. Answer: Amount to be paid for Delay in Furnishing Return Rule 7C of STR, 1994 Where the return prescribed under rule 7 is furnished after the date prescribed for submission of such return, the person liable to furnish the said return shall pay to the credit of the Central Government, for the period of delay of(i) 15 days from the date prescribed for submission of such return, an amount of `500. (ii) beyond 15 days but not later than 30 days from the date prescribed for submission of such return, an amount of `1,000; and (iii) beyond 30 days from the date prescribed for submission of such return an amount of `1,000 plus `100 for every day from the thirty first day till the date of furnishing the said return.

Service Tax

30

Provided that the total amount payable in terms of this rule, for delayed submission of return, shall not exceed `20,000. Provided also that where the gross amount of service tax payable is nil, the Central Excise Officer may, on being satisfied that there is sufficient reason for not filing the return, reduce or waive the penalty. Question 35: Write a note on contents of the Service Tax Return. Answer: Contents of the Return 1. Half year for which return is being filed 2. Name of the assessee 3. STC Number 4. Address 5. Constitution of the assessee like individual or partnership firm etc. 6. Assessee liable to pay service tax as service provider or service receiver under reverse charge 7. Gross amount received 8. Amount of service tax and education cess 9. Tax credit allowed 10. Net tax payable 11. Details of services rendered 12. Verification of the return 13. Signature of service provider or service recipient under reverse charge Question 36: Write a note on documents to be submitted along with Service Tax Return. Answer: Documents to be submitted along with the return Alongwith ST-3 return following documents should be attached: (i) copies of GAR-7 challans which indicate the payment of service tax. (ii) a memorandum in form ST-3A giving details of the difference between the amount of provisional tax deposited and the actual amount payable for each month. Every service tax assessee should furnish to the Superintendent of Central Excise, at the time of filing first half yearly return, a list of all accounts maintained by him in relation to service tax. The service tax return should be filed in triplicate to the Superintendent of Central Excise . The assessee may also file the return with the concerned Divisional Office by registered post However, the assesses should ensure that the return reaches the Divisional Office on or before the due date. For an assessee who provides more than one taxable service, filing of a single return is sufficient . However, the details in each of the columns of the Form ST-3 have to be furnished separately for each of the taxable service rendered by him. Even if no service has been provided during a half year and no service tax is payable; the assessee has to file a Nil return within the prescribed time limit. Question 37 (V. Imp.): Write a note on E-Filing of Service Tax Return. Answer: E-Filing of Return With effect from 01.10.2011, e-filing of service tax returns has been made mandatory for all the assesses (Notification No. 43/2011 dated: 25.08.2011). The assessee can e-file the return through software ACES i.e. AUTOMATION OF CENTRAL EXCISE AND SERVICE TAX.

Service Tax

31

The assessee should login the relevant site www.aces.gov.in. To transact business on ACES a user has to first register himself with ACES through a process called Registration with ACES. The assessee should have STP code i.e. PAN based 15 digit registration number allotted by Service Tax Department. The assessee should fill in self-chosen user-ID and his e-mail ID and system will check the availability of the chosen user ID and then generate a password and will send it to the assessee on the e-mail ID mentioned by him and the assessee can submit the return in the prescribed form. Benefits of E-Filing for assesee: 1) Reduce Physical Interface with the Department; 2) Save Time; 3) Reduce Paper Work; 4) Online Registration and Amendment of Registration; 5) Electronic filing of all documents such as applications for registration, returns, claims, permissions and intimations; export-related documents, refund request; 6) System-generated E-Acknowledgement; 7) Online tracking of the status of selected documents. Question 38: Write a note on Service Tax Return Preparer Scheme. Answer: Service Tax Return Preparer Scheme Section 71 Department shall appoint Service Tax Return Preparer to assist the persons to file service tax return. The STRP shall have required qualification i.e. he should have passed senior secondary level examination. The Service Tax Return Preparer shall (a) (b) (c) (d) (e) prepare the return with due diligence; affix his signature on the return prepared by him; furnish the return as specified; hand over a copy of the return to the person whose return is prepared and furnished by him; retain a copy of the acknowledgment of having furnished the return

In respect of returns prepared and furnished by him maintain record of the following, namely:(i) the name of assessees whose returns have been prepared and furnished by him during that month; (ii) the Service Tax Code(STC) number and premises code of such assessees; (iii) period for which return is filed; (iv) date of furnishing the return; (v) authority with whom return is filed; (vi) amount of tax payable; (vii) amount of tax paid; (viii) the fee charged and received by him Incentive to Service Tax Return Preparers An assessee shall pay a fee as may be mutually agreed upon between an assessee and the Service Tax Return Preparer. The Board recommends, as a yardstick, a fee of rupees one thousand per return prepared by the Service Tax Return Preparer. Question 39: Write a note on Records under Service Tax. Answer: Records Rule 5 of STR, 1994 The records including computerised data, as maintained by an assessee in accordance with the various laws in force from time to time shall be acceptable. All such records shall be preserved at least for a period of 5 years immediately after the financial year to which such records pertain. Every assessee shall make available, at the registered premises, at all reasonable time, all such records, for inspection and examination by the Central Excise Officer authorised in writing by the jurisdictional Assistant Commissioner or Deputy Commissioner of Central Excise, as the case may be.

Service Tax

32

Explanation. - For the purposes of this rule, registered premises includes all premises or offices from where an assessee is providing taxable services. As per section 77, any person who has failed to maintain the book of accounts shall pay penalty which may extend upto `10,000. Question 40 (V. Imp.): Write a note on option to pay service tax in case of Air Travel Agent. Answer: Option to pay service tax in case of Air Travel Agent Rule 6(7) The person liable for paying the service tax in relation to the services of booking of tickets for travel by air provided by an air travel agent, shall have the option, to pay an amount at the rate of 0.6% plus EC of the basic fare in the case of domestic bookings, and at the rate of 1.2% plus EC of the basic fare in the case of international bookings, of passage for travel by air, during any calendar month or quarter, as the case may be, towards the discharge of his service tax liability instead of paying service tax at the normal rate and the option, once exercised, shall apply uniformly in respect of all the bookings of passage for travel by air made by him and shall not be changed during a financial year under any circumstances. Basic Fare means that part of the air fare on which commission is normally paid to the air travel agent by the airline. Illustration 2: Mr. Anurag is an Air Travel Agent and provides the following information for the month of October, 2012. Date 02/10/2012 10/10/2012 24/10/2012 Particulars Booked by Mr. A for Delhi to Chennai Booked by Mr. Somnath for Delhi to Bangkok Booked by Mr. Raj for Delhi to Bhubaneswar Basic Fare 3,000 15,000 2,600 Taxes Other than Service tax 2,500 2,500 2,400 Commission 500 1,500 500 Total Value 6,000 19,000 5,500

Compute service tax payable by Mr. Anurag under various methods applicable for air travel agent. Solution: Method 1 Total Commission 2,500.00 Service tax @ 12.36% 309.00 Method 2 As per Rule 6(7) of Service Tax Rules 1994 02.10.2012 3,000 x 0.618% 10.10.2012 15,000 x 1.236% 24.10.2012 2,600 x 0.618% Total Service Tax Payable Rounded off u/s 37D ` 18.54 185.40 16.07 220.01 220.00

Question 41 (V. Imp.): Write a note on option to pay service tax by any insurer carrying on Life Insurance Business. Answer:

Service Tax

33

Option to pay service tax by any insurer carrying on life insurance business Rule 6(7A) An insurer carrying on life insurance business shall have the option to pay tax: (i) on the gross premium charged from a policy holder reduced by the amount allocated for investment, or savings on behalf of policy holder, if such amount is intimated to the policy holder at the time of providing of service; (ii) the service provider shall pay service tax @ 3% on the amount of gross premium for the first year and 1.5% for the subsequent years Provided that such option shall not be available in cases where the entire premium paid by the policy holder is only towards risk cover in life insurance. Illustration 3: Mr. X is taken a policy from LIC and premium payable is `50,000 per annum (including Risk Premium of `10,000). Calculate the service tax to be charged by LIC under various methods. Solution: Method 1 Risk Premium x service tax rate 10,000 x 12.36% = `1,236 Method 2 Gross Amount charged x 3.09% 50,000 x 3.09% = `1,545 Question 42 (Imp.): Write a note on option to pay service tax in case of Money Changer. Answer: Option to pay service tax in case of a Money Changer Rule 6(7B) The person liable to pay service tax in relation to purchase or sale of foreign currency, including money changing, shall have the option to pay an amount calculated at the following rate towards discharge of his service tax liability instead of paying service tax at the normal rate: (a) 0.12% of the gross amount of currency exchanged upto ` 1,00,000, but minimum ` 30; (b) on next 9,00,000 @ 0.06% (c) on balance @ 0.012% but maximum `6,000. Provided that the person providing the service shall exercise such option for a financial year and such option shall not be withdrawn during the remaining part of that financial year. Alternatively, the assessee can find value of service as provided in rule 2B of Service Tax (Determination of Value) Rules, 2006. As per these rules, the value of service will be equal to difference in buying or selling rate (as the case may be) and the RBI reference rate for the currency at that time. [RBI Reference Rate is mean of selling and buying rate]. Example I: US$1000 are sold by a customer at the rate of Rupees 45 per US$. RBI reference rate for US$ is Rupees 45.50 for that day. The taxable value shall be Rupees 500.

Service Tax

34

Example II: INR70000 is changed into Great Britain Pound (GBP) and the exchange rate offered is Rupees 70, thereby giving GBP 1000. RBI reference rate for that day for GBP is Rupees 69. The taxable value shall be Rupees 1000. Illustration 4: Mr. X is a dealer and engaged in sale & purchase of Foreign currency. ABC Ltd requires 10000 US Dollar to make foreign payment. Mr. X quotes ` 51 per US Dollar and RBI reference rate is `50 per dollar. Calculate the amount of service tax under different option: Solution: Option 1 10,000 (51-50) x 12.36% Option 2 Gross Amount Charged 51 x 10,000 Service Tax Up to 1,00,000 Next 4,10,000 Education cess @ 3% Service tax payable Rounded Off u/s 37D ` 1,236 5,10,000

1,00,000 x 0.12% 4,10,000 x 0.06%

120.00 246.00 366.00 10.98 376.98 377.00

Question 43: Write a note on option to pay service tax in case of Distribution of Lottery ticket. Answer: Option to pay service tax in case of distributor of lottery tickets Rule 6(7C) The distributor or selling agent, liable to pay service tax for the taxable service of promotion, marketing, organising or in any other manner assisting in organising lottery, shall have the option to pay an amount at the rate mentioned below: Sl. No. (1) 1. Rate (2) ` 7,000 on every ` 10 lakh (or part of ` 10 lakh) of aggregate face value of lottery tickets printed by the organising State for a draw ` 11,000 on every ` 10 lakh (or part of ` 10 lakh) of aggregate face value of lottery tickets printed by the organising State for a draw Condition (3) If the lottery or lottery scheme is one where the guaranteed prize payout is more than 80% If the lottery or lottery scheme is one where the guaranteed prize payout is less than 80% :

2.

In case of online lottery, the aggregate face value of lottery tickets shall be taken as the aggregate value of tickets sold. The distributor or selling agent shall exercise such option within a period of one month of the beginning of each financial year and such option shall not be withdrawn during the remaining part of the financial year.

Service Tax

35

Illustration 5: M/s Future Gaming Solutions India Private Limited is a distributor of lottery organized by State of Sikkim provides following information: Particulars Diwali Bumper Total No of Tickets 2,50,000 Face Value of Tickets 100 Value of Guaranteed Prize Payout 51.11% Actual Number of ticket sold 2,00,000 Mode of conducting the scheme Printed Compute Service tax payable under Rule 6(7C) of service tax Rules, 1994. Solution: ` Printed Mode 2,50,000 x 100 x 11,000 = 10,00,000 Education Cess @ 3% On Line Mode 2,00,000 x 100 x 11,000 10,00,000 Education Cess @ 3% 2,75,000 8,250 2,83,250 2,20,000 6,600 2,26,600 Diwali Dhamaka 2,50,000 100 51.11% 2,00,000 On line

Illustration 6: Mr. Amit Rastogi is a distributor of lotteries organized by the State of Haryana. He is running two schemes of lotteries as followedTotal No. of ticket proposed under the scheme Face value per ticket Value of guaranteed prize payouts Actual no. of tickets sold Mode of conducting the Scheme Scheme A 22,75,000 10 75% 17,50,000 Printed 22,75,000 Scheme B 50,000 250 85% 46,250 Online

Compute service tax payable under Rule 6(7C) of Service Tax Rule, 1994. Solution: Computation of Service Tax Liability of Mr. Amit Rastogi Statement showing computation of Service Tax liability under Composition Scheme (Rule 6(7C) of the STR, 1994) of Mr. Amit Rastogi. Particulars Scheme A Scheme B Total no. of tickets 22,75,000 46,250 Face value per ticket 10 250 Aggregate value of lottery tickets 2,27,50,000 1,15,62,500 Value of guaranteed prize payouts 75% 85% No. of units of 10 lakhs or part thereof 23 12 Service tax payable(for every 10 lakhs or part thereof) 11,000 7,000 Service tax payable 2,53,000 84,000

Service Tax Total service tax Add: EC @ 2% Add: SHEC @ 1% Total service tax liability

36 3,37,000 6,740 3,370 3,47,110

Question 44 (V. Imp): Write a note on tax credit in case of Service Provider. Answer: A Service provider shall be allowed tax credit for the following tax paid by him. 1. Excise duty on inputs (raw material etc) or capital goods (plant and machinery, furniture and fixtures etc) used in or in connection with providing of output services. 2. Service Tax paid on the input services taken by him in connection with providing of output services and such input services may be insurance services, banking services, renting of immovable property or other similar services. 3. Education cess and SHEC paid on Excise Duty or Service Tax shall also be eligible for tax credit. The service provider can utilize the tax credit in the manner given below: 1. Tax credit for Excise Duty or Service Tax can be utilized against output service tax. 2. Tax credit for Education cess of 2% can be utilized for payment of education cess of 2% on output service tax. 3. Tax credit for SHEC of 1% can be utilized for payment of SHEC of 1% on output service tax. Inter-adjustment of Tax credit of Excise Duty, Service Tax is allowed to the service provider because all these taxes are collected by Central Government. Tax credit for Excise Duty, Service Tax is called Cenvat Credit and is regulated through Cenvat Credit Rules, 2004. Question 45: Whether any section of Central Excise Act, 1944 is applicable to Service tax. (For Self Reading Only) Answer: Application of certain provisions of Central Excise Act 1944 Section 83 The following sections of the Central Excise Act, 1944, shall apply to service tax as they apply in relation to a duty of excise: 9A, 9AA, 9B, 9C, 9D, 9E, 11B, 11BB, 11C, 12, 12A, 12B, 12C, 12D, 12E, 14, 15, 31, 32, 32A to 32P (both inclusive), 33A, 34A, 35EE, 35F, 35FF to 35-O (both inclusive), 35Q, 35R, 36, 36A, 36B, 37A, 37B, 37C, 37D, 38A and 40. Question 46 (V. Imp.): Write a note on valuation of taxable services for charging Service Tax. Answer: Valuation of taxable services for charging service tax Section 67 Value shall be determined in the manner given below: (a) If consideration is in terms of money, the money value of the services e.g. If a TV Programme Production services charges `5,00,000 (exclusive of service tax) as fee from its client, the value of the taxable service rendered will be `5,00,000 and service tax payable shall be `61,800 (5,00,000 x 12.36%).

Service Tax

37

Where the gross amount charged by a service provider, for the service provided or to be provided is inclusive of service tax payable, the value of such taxable service shall be such amount as, with the addition of tax payable, is equal to the gross amount charged. E.g. If total amount charged is `5,00,000 and service tax has not been charged separately, in that case amount of service tax shall be 5,00,000 / 112.36 x 12.36 = `55,001.77 Rounded Off `55,002 (b) If consideration is in kind, market value of such consideration and it will be considered to be inclusive of service tax. E.g. One service provider has rendered services and service recipient has given one gold watch with market value `50,000, in this case amount of service tax shall be 50,000 / 112.36 x 12.36 = `5,500.18 rounded off `5,500 (c) If consideration is partly in cash and partly in kind, it will be total of cash plus market value of consideration in kind and it will be considered to be inclusive of service tax. e.g. A Survey and Map Making Agency provides taxable professional services to one of its clients and it charges `1,00,000 in cash and `75,000 in kind, in this case total consideration shall be `1,75,000 but it will be inclusive of service tax and accordingly amount of service tax shall be 12.36/ 112.36 x 1,75,000 = `19,250.62 Rounded off u/s 37D = `19,251 (d) If any person has rendered free services, no service tax is payable. Question 47: Explain the concept of Pure Agent. Answer: As per Service Tax (Determination of Value) Rules, 2006, Pure agent means a person who (a) Enters into a contractual agreement with the recipient of service to act as his pure agent to incur expenditure or costs in the course of providing taxable service; (b) do not hold any title to the goods or services so procured as pure agent of the recipient of service; (c) does not use such goods or services so procured; and (d) receives only the actual amount incurred to procure such goods or services. The expenditure or costs incurred by the service provider as a pure agent of the recipient of service, shall be excluded from the value of the taxable service. If the service provider is not a pure agent, the value of the taxable service is the total amount of consideration consisting of all components of the taxable service and it is immaterial that the details of individual components of the total consideration is indicated separately in the invoice. Illustration. X contracts with Y, a real estate agent to sell his house and thereupon Y gives an advertisement in television. Y billed X including charges for television advertisement and service tax paid on advertisement in the total bill. In such a case, consideration for the service provided is what X pays to Y. Y does not act as an agent on behalf of X when obtaining the television advertisement even if the cost of

Service Tax

38

television advertisement is mentioned separately in the invoice issued by Y. Advertising service is an input service for the estate agent in order to enable or facilitate him to perform his services as an estate agent Illustration. In the course of providing a taxable service, a service provider incurs costs such as travelling expenses, postage, telephone, etc., and may indicate these items separately on the invoice issued to the recipient of service. In such a case, the service provider is not acting as an agent of the recipient of service but procures such inputs or input service on his own account for providing the taxable service. Such expenses do not become reimbursable expenditure merely because they are indicated separately in the invoice issued by the service provider to the recipient of service and thus shall form part of value of taxable service. Illustration. A contracts with B, an architect for building a house. During the course of providing the taxable service, B incurs expenses such as telephone charges, air travel tickets, hotel accommodation, etc., to enable him to effectively perform the provision of services to A. In such a case, in whatever form B recovers such expenditure from A, whether as a separately itemised expense or as part of an inclusive overall fee, service tax is payable on the total amount charged by B. Value of the taxable service for charging service tax is what A pays to B. Illustration. Company X provides a taxable service of rent-a-cab by providing chauffeur-driven cars for overseas visitors. The chauffeur is given a lump sum amount to cover his food and overnight accommodation and any other incidental expenses such as parking fees by the Company X during the tour. At the end of the tour, the chauffeur returns the balance of the amount with a statement of his expenses and the relevant bills. Company X charges these amounts from the recipients of service. The cost incurred by the chauffeur and billed to the recipient of service constitutes part of gross amount charged for the provision of services by the company X. Question 48 (V. Imp.): Explain meaning of Input Service / Output Service. Answer: Input service Rule 2(l) of Cenvat Credit Rules, 2004 means any service, (i) used by a provider of outpur service for providing an output service; or (ii) used by a manufacturer, whether directly or indirectly, in or in relation to the manufacture of final products and clearance of final products upto the place of removal, and includes services used in relation to modernisation, renovation or repairs of a factory, premises of provider of output service or an office relating to such factory or premises, advertisement or sales promotion, market research, storage upto the place of removal, procurement of inputs, accounting, auditing, financing, recruitment and quality control, coaching and training, computer networking, credit rating, share registry, security, business exhibition, legal services, inward transportation of inputs or capital goods and outward transportation upto the place of removal.

Output service

Rule 2(p) of Cenvat credit Rules, 2004

means any service provided by a provider of service located in the taxable territory but shall not include a service,

Service Tax (1) specified in section 66D of the Finance Act; or (2) where the whole of service tax is liable to be paid by the recipient of service.

39

Question 49 (Imp.): Write a note on Input Service Distributor. Answer: Input service distributor Rule 2(m) of Cenvat Credit Rules, 2004 means an office of the manufacturer or provider of output service, which receives invoices towards purchases of input services and issues invoice, for the purposes of distributing the credit of service tax paid on the said services to its manufacturing units or units providing output service. The ISD must be registered with Service Tax Department. Credit of service tax attributable to service used in a unit exclusively engaged in manufacture of exempted goods or providing of exempted services shall not be distributed. The credit so distributed should not exceed the amount of service tax paid on input services. Every input service distributor distributing credit shall, issue an invoice, signed by such person for each of the recipient of the credit distributed, and such invoice, shall be serially numbered and shall contain the following namely:(i) the name, address and registration number of the person providing input services and the serial number and date of invoice; (ii) the name and address of the input service distributor; (iii)the name and address of the recipient of the credit distributed; (iv) the amount of the credit distributed. Question 50: Explain provisions of Tax Deduction at Source (TDS) with regard to Service Tax. Answer: If any service recipient has to make the payment to the service provider and tax has to be deducted at source, in that case, while deducting tax at source, service tax shall also be included in some of the cases and it will not be included in some other cases and is as given below: TDS excluding Service Tax 1. If the service recipient is making payment of Rent, tax shall be deducted at source under section 194-I @ 10% and tax shall be deducted excluding service tax. Example ABC Ltd. has let out one commercial building to XYZ Ltd. and rent charged is `20,00,000 plus service tax, in this case, tax to be deducted at source by XYZ Ltd. shall be 20,00,000 x 10% = `2,00,000 and amount payable to ABC Ltd. shall be Rent 20,00,000 Add: Service Tax @ 12.36% 2,47,200 Total 22,47,200 Less: TDS (20,00,000 x 10%) 2,00,000 Amount Payable 20,47,200

Service Tax

40

2. If the service recipient is making payment of Commission / Brokerage, tax shall be deducted at source under section 194-H @ 10% and tax shall be deducted excluding service tax. Example Mr. X a property dealer has rendered services for getting one commercial building on rent to XYZ Ltd. and Commission charged is `5,00,000 plus service tax, in this case, tax to be deducted at source by XYZ Ltd. shall be 5,00,000 x 10% = `50,000 and amount payable to Mr. X shall be Commission 5,00,000 Add: Service Tax @ 12.36% 61,800 Total 5,61,800 Less: TDS (5,00,000 x 10%) 50,000 Amount Payable 5,11,800 3. If the service recipient is making payment of Commission for Sale of Lottery Tickets , tax shall be deducted at source under section 194-G @ 10% and tax shall be deducted excluding service tax. ABC Ltd. has appointed Mr. X as a salesman for sale of lottery tickets and Mr. X has charged commission of `2,00,000 plus service tax @ 12.36%, in this case, amount of TDS shall be Commission 2,00,000 Add: Service Tax @ 12.36% 24,720 Total 2,24,720 Less: TDS (2,00,000 x 10%) 20,000 Amount Payable 2,04,720 4. If the service recipient is making payment of Commission for Insurance Business, tax shall be deducted at source under section 194-D @ 10% and tax shall be deducted excluding service tax. There is a Reverse Charge and service tax has to be paid by the Insurance Company (Such services are called Insurance Auxiliary Services) Example If LIC has to commission of `2,00,000 to their agent Mr. X in connection with insurance business, amount of TDS shall be `2,00,000 x 10% = `20,000 and service tax of `24,720 shall be paid directly by LIC to the Government under Reverse Charge under Rule 2(1)(d). TDS including Service Tax 1. If the service recipient is making payment for Professional / Technical services, tax shall be deducted at source under section 194-J @ 10% and tax shall be deducted including service tax. Example Mr. X, a Chartered Accountant has given professional services to XYZ Ltd. and charged `20,00,000 plus service tax, in this case, tax to be deducted at source by XYZ Ltd. shall be 22,47,200 x 10% = `2,24,720 and amount payable to Mr. X shall be Professional Charges 20,00,000 Add: Service Tax @ 12.36% 2,47,200 Total 22,47,200 Less: TDS (22,47,200 x 10%) 2,24,720 Amount Payable 20,22,480

Service Tax

41

2. If the service recipient is making payment for any Contract e.g. Advertising Contract or Catering Contract etc tax shall be deducted at source under section 194-C @ 2% but if payment is being given to any individual or HUF, tax shall be deducted at source @ 1% and tax shall be deducted including service tax. Example ABC Ltd. has rendered advertising services to XYZ Ltd. and charged `20,00,000 plus service tax, in this case, tax to be deducted at source by XYZ Ltd. shall be 22,47,200 x 2% = `44,944 and amount payable to ABC Ltd. shall be Advertising Charges 20,00,000 Add: Service Tax @ 12.36% 2,47,200 Total 22,47,200 Less: TDS (22,47,200 x 2%) 44,944 Amount Payable 22,02,256 Date of determination of rate of tax, value of taxable service and rate of exchange Section 67A The rate of service tax, value of a taxable service and rate of exchange, if any, shall be the rate of service tax or value of a taxable service or rate of exchange, as the case may be, in force or as applicable at the time when the taxable service has been provided or agreed to be provided. Explanation. For the purposes of this section, "rate of exchange" means the rate of exchange referred to in the Explanation to section 14 of the Customs Act, 1962.'; Applicable from 28.05.2012 Illustration 7: Determine POT in the following cases: Date of completion 01.08.2012 01.07.2012 01.07.2012 01.08.2012 Solution: Date of completion 01.08.2012 01.07.2012 01.07.2012 01.08.2012 Invoice Date 20.08.2012 20.08.2012 20.07.2012 31.08.2012 Payment Date 18.08.2012 18.08.2012 01.05.2012 01.09.2012 POT Date 18.08.2012 01.07.2012 01.05.2012 31.08.2012 Invoice Date 20.08.2012 20.08.2012 20.07.2012 31.08.2012 Payment Date 18.08.2012 18.08.2012 01.05.2012 01.09.2012

Illustration 8: Mr. X started rendering services w.e.f. 01.04.2012 and has submitted particulars as given below:1) 2) 3) Rendered services on 10.04.2012, bill issued on 20.04.2012 amount `10 Lakh, Payment Received on 10.11.2012 Rendered services on 06.07.2012 `60 Lakh and issued bill on 01.08.2012 and payment received on 10.12.2012 Rendered services on 07.11.2012 `30 Lakh, bill issued on 08.11.2012 and payment received in advance on 03.10.2012.

Service Tax 4)

42

Rendered services on 10.03.2013 `10 Lakh, bill issued on 01.04.2013 and payment received on 03.04.2013

Service Tax has been charged separately, wherever it was applicable. Compute Service Tax Payable for each quarter and the last date up to which the payment should be made. Solution: Calculation of Service Tax Payable First Quarter (Apr-June) In this Quarter the service provider has started rendering services w.e.f. 01.04.2012, hence no service tax is payable on first `10 Lakh Second Quarter (July-sep.) Total bills issued are ` 60 lakh, out of which service of the value of ` 50 Lakh shall be payable on Receipt basis. (i.e. In the third quarter) Service tax on balance `10 Lakh shall be payable as per rule 3 of Point of Taxation Rule 2011 and Point of Taxation shall be 01.08.2012 ` `10,00,000 x 12.36% 1,23,600 Service Tax Payable 1,23,600 3rd Quarter (Oct. to Dec.) `50,00,000 x 12.36% 6,18,000 `30,00,000 x 12.36% 3,70,800 Total Service Tax Payable 9,88,800 4th Quarter (Jan to March) Since Point of taxation is 01.04.2013 no service tax is payable. Last date of the payment of service tax for: 1st Quarter 2nd Quarter 3rd Quarter 4th Quarter 5th July/6th July 5th Oct/6th Oct 5th Jan/6th Jan. 31st March

Vous aimerez peut-être aussi